1959 AHSME Problems/Problem 7

Revision as of 05:56, 24 September 2018 by Dajeff (talk | contribs) (Solution)

Solution

If we let $a=3$ and $d=1$, then we will get a $3$-$4$-$5$ triangle, which is a right triangle. So, the answer is $\boxed{\textbf{(D)}3:1}$.